Evaluating a Path Integral: x^2+y^2+z^2

Click For Summary
SUMMARY

The discussion focuses on evaluating the path integral of the function \(x^2 + y^2 + z^2\) along a specified path from point \(a = (0,0,0)\) to point \(b = (3,4,5)\). The user initially struggled with the notation and concepts but clarified that \(dr\) represents the differential vector along the path. The solution involves recognizing the parametric representation of the path and applying the concept of spherical symmetry, where \(r^2 = x^2 + y^2 + z^2\).

PREREQUISITES
  • Understanding of path integrals in vector calculus
  • Familiarity with parametric equations
  • Knowledge of spherical coordinates and symmetry
  • Basic proficiency in calculus, particularly calculus III concepts
NEXT STEPS
  • Study the evaluation of path integrals in vector calculus
  • Learn about parametric equations and their applications in integrals
  • Explore spherical coordinates and their use in multivariable calculus
  • Review the concept of vector fields and line integrals
USEFUL FOR

Students and educators in mathematics, particularly those studying calculus and vector analysis, as well as anyone seeking to deepen their understanding of path integrals and their applications in physics and engineering.

jinksys
Messages
122
Reaction score
0

Homework Statement



Evaluate the path integral \int (x^2+y^2+z^2)dr from a =(0,0,0) to b= (3,4,5).

Homework Equations


The Attempt at a Solution



I'm lost. Had the dr been a ds I could do it, but my calculus book only deals with situations where \int F.dr.Edit: I figured it out, it's been a while since I've had calc 3.

I had forgotten what r represented. r = xx^ + yy^ +zz^.
You then multiply dr by x^2 + y^2 + z^2 and go parametric.
 
Last edited:
Physics news on Phys.org
is the path a straight line?

as it may also help to note the spherical symmetry r^2 = x^2 + y^2 + z^2
 
Question: A clock's minute hand has length 4 and its hour hand has length 3. What is the distance between the tips at the moment when it is increasing most rapidly?(Putnam Exam Question) Answer: Making assumption that both the hands moves at constant angular velocities, the answer is ## \sqrt{7} .## But don't you think this assumption is somewhat doubtful and wrong?

Similar threads

  • · Replies 3 ·
Replies
3
Views
2K
  • · Replies 19 ·
Replies
19
Views
2K
Replies
14
Views
4K
  • · Replies 2 ·
Replies
2
Views
1K
  • · Replies 9 ·
Replies
9
Views
2K
  • · Replies 3 ·
Replies
3
Views
2K
  • · Replies 2 ·
Replies
2
Views
1K
Replies
3
Views
2K
Replies
7
Views
2K
  • · Replies 21 ·
Replies
21
Views
3K